Mayor: Local antitobacco activists are calling for expanded antismoking education programs paid for by revenue from ...

filozinni on May 24, 2020

Answer C

Why is C the right answer? Thank you

Replies
Create a free account to read and take part in forum discussions.

Already have an account? log in

Skylar on May 24, 2020

@filozinni, happy to help!

The argument made by this passage is that "the taxes themselves would produce the sought-after reduction in smoking." We are asked to select the answer choice that "most undermines" this reasoning.

(C) "Usually, cigarette sales will increase substantially in the areas surrounding a city after that city imposes stiff taxes on cigarettes."

This is the correct answer choice because it undermines the idea that there would be a reduction in smoking. (C) suggests that the taxes will merely have the effect of changing the location in which people purchase cigarettes from inside the city to outside the city. This shift in purchase location does not mean there is an overall reduction in smoking, so (C) undermines the passage's reasoning and is the best answer.

Does that make sense? Please let us know if you have other questions or need clarification on why any of the other answer choices are incorrect!

filozinni on June 3, 2020

Oh I understand now! Thank you so much @Skylar!

Lenny on June 6, 2021

I don't understand. The question repeatedly states "in the city" so I crossed out this answer choice as being out of scope. Why did they particularly state "in the city" twice when they wanted an answer that undermined the purchase of tobacco products outside the city?

on March 23, 2022

Yes I was having the same issue with this question, I immediately crossed out this answer choice based on the wording. The stem repeatedly mentions "the city" but the answer choice C states that areas outside the city continued to have high tobacco sales.

Emil-Kunkin on March 25, 2022

While the Mayor is discussing sales within the city, their goal would be to actually reduce smoking. C tells us that the tax might fail to reduce that goal, even if sales decrease in the city, they might rise in the nearby areas. That is, people from the city will simply go just outside the city to buy tobacco. This means that the program will not have the desired effect of cutting smoking, because smokers will simply buy their cigarettes in areas near the city. For strengthen and weaken questions, the answer choices can be a bit broader in scope as long as it actually strengthens or weakens the argument.